Answer:
7 boxes
Step-by-step explanation:
We can use ratios to solve
12 bars 42 bars
---------- = ---------------
2 boxes x boxes
Using cross products
12x = 2 *42
12x = 84
Divide by 12
12x/12 = 84/12
x = 7
Rectangle ABCD translates 4 units down and 2 units to the right to form rectangle A'B'C'D'. The vertices of rectangle ABCD are labeled in alphabetical order going clockwise around the figure. If AB = 3 units and AD = 5 units, what is the length of B'C'?
Answer:
The length of BC is 14 units.Step-by-step explanation:
[tex]hope \: \: it \: \: helps} \beta \alpha \infty [/tex]
The length of B'C' is 0 units.
What is translation?It is the movement of the shape in left, right, up, and down direction.
The translated shape will have the same shape and shape.
There is a positive value when translated to the right and up.
There is a negative value when translated to the left and down.
We have,
The length of AD = 5 units.
Since the rectangle translates down by 4 units,
The length of A'D' =5 units.
The width of the original rectangle is AB, which is 3 units.
Since the rectangle translates to the right by 2 units,
The width of the new rectangle = 3 units.
Now,
The length of B'C' is the same as the length of AD', which is 5 units.
Subtracting 5 units from 5 units gives us a length of 0 units.
Thus,
The length of B'C' is 0 units.
Learn more about translation here:
https://brainly.com/question/12463306
#SPJ7
ASAP there are three marbles in a bag. One is red and two are black. What is the probability of picking a black marble first, putting it back in the bag and then picking a black marble? Use the following probability to find the answer.
Answer:
[tex] \frac{4}{9} [/tex]
Step-by-step explanation:
[tex]p = \frac{favorable \: outcomes}{total \: outcomes} = \frac{4}{9} [/tex]
=============================================================
Explanation:
The probability you get a black marble on the first selection is 2/3 since we have 2 black marbles out of 2+1 = 3 total.
We put the marble back and then we have 2/3 as the probability of selecting another black marble on the second try. Nothing has changed because we put the marble back. That means the events are independent.
So we get (2/3)*(2/3) = 4/9 as the probability of selecting 2 black marbles in a row (with replacement).
Six consecutive numbers add up to a total of 69.what is the highest of these numbers?
Answer:
14
Step-by-step explanation:
9+10+11+12+13+14=69
The [tex]HIGHEST[/tex] of these numbers is [tex]14[/tex]
Let the [tex]CONSECUTIVE[/tex] numbers be : [tex]a, a+1, a +2,a+3,a+4,a+5[/tex]
Taking the sum :
[tex]a + a + 1 + a + 2 + a + 3 + a + 4 + a + 5 = 69\\6a + 15 = 69\\6a = 69 - 15\\6a = 54\\a = 54 / 6\\a = 9[/tex]
[tex]HIGHEST[/tex] value = [tex]a + 5 = 9 + 5 = 14[/tex]
Hence, [tex]HIGHEST[/tex] value = [tex]14[/tex]
Learn more : https://brainly.com/question/15974141
Which diagram represents the hypothesis of the converse of corresponding angles theorem?
Answer:
the first diagram
Step-by-step explanation:
first one
The ice cream man just ended his shift for the day. Let 1/2x^2 6/11x + 8 represent the amount of chocolate ice cream bars he sold. Let 5/9x^2 + 2/3 represent the amount of vanilla ice cream bars he sold. Finally let 1/3x^2 + 4x + 4/3 represent the amount of strawberry ice cream bars he sold. Select all the statements that are true
a. The total amount of ice cream bars sold can be represented by the expression 25/18x^2+ 50/11x +10
b. The total amount of ice cream bars sold can be represented by the expression 25/18x^2 + 172/33x +28/3
c. He sold 1/6x^2 + 50/11x + 28/3 more chocolate than strawberry ice cream bars.
d. He sold 1/6x^2 - 38/11x + 20/3 more chocolate than strawberry ice cream bars.
Answer:
A and D
Step-by-step explanation:
Total ice cream bars sold = sum of chocolate sold , vanilla and strawberry ice-creams sold.
=(1/2)x2 + (6/11)x + 8 + (5/9)x2 + (2/3) +(1/3)x2 + 4x +(4/3) (Given in the question)
=(25/18)x2 + (50/11)x + 10 (Adding terms corresponding to x2,x ,constant respectively)
Difference in chocolate and strawberry bars =[ (1/2)x2 + (6/11)x + 8] - [(1/3)x2 + 4x +(4/3)]
= (1/6)x2 - (38/11)x +(20/3)
So, the correct options are A and D
Ibrahim heeft een bijbaantje op de markt. Hij berekent zijn inkomsten met de formule
inkomsten in €=5+3,50 x tijd in uren. Leg de formule uit.
Answer:
Ibrahim gets 5 fixed and 3.5 per hour.
Step-by-step explanation:
Ibrahim has a side job at the market. He calculates his income with the formula income in € = 5 + 3.50 x, time in hours. Explain the formula.
Here, the fixed income is 5.
the income per hour is 3.5.
So, Ibrahim gets 5 fixed and 3.5 per hour.
Suppose that from a group of 9 men, 1 will be randomly chosen for a dangerous assignment, and suppose that the chosen man will be killed during the assignment with a probability of 1/6. If Mark is one of the 9 men, what is the probability that he will be chosen for the assignment and killed during the assignment
Answer:
1/54
Step-by-step explanation:
1/9 x 1/6
 Marsha has a bag that contains 4 green marbles, 8 yellow marbles , and 20 red marbles . If she chooses one marble from the bag, what is the probability that the marble is not red?
PLEASE HELP IF YOURE GOOD AT GEOMETRY!!
Answer:
C. 3/8
HOPE THIS HELPS :)
Answer:
c. 3/8
Step-by-step explanation:
first you need the denomerator by adding all marbles together which equals 32. now for the munerator you need the sum of the green and yellow marbles. this equals 12. so your fraction is 12/32. next we simplify. we can divide both numbers by 4. getting us a fraction of 3/8.
Help Me Pls i need it now
Nonsense = Report
Answer:
8,6,3, v= 144
4,8,6, v=192
15,10,6, v=900
Step-by-step explanation:
Answer:
This geometric questions are very very simple let's start to solve all steps
Step-by-step explanation:
L means long of Prism and look at 8 and 6 for first prism. Which one is longest of course 8
w means wide =6
h means high=3 and
V means Volume: You must multiply by 3, 6,8 to find volume, so we can say Volume 3*6*8=144 easily
0.7(1.5 + y) = 3.5y - 1.47
Answer:
y = 0.9
Step-by-step explanation:
1.05 + 0.7y = 3.5y - 1.47
-3.5y + 0.7y = -1.47 - 1.05
-2.8y = -2.52
y = 9/10 = 0.9
Answer:
[tex]\textbf{HELLO!!}[/tex]
[tex]0.7\left(1.5+y\right)=3.5y-1.47[/tex]
[tex]1.05+0.7y=3.5y-1.47 \gets \textsl{Expand}[/tex]
[tex]1.05+0.7y-1.05=3.5y-1.47-1.05 \gets Subtract\; 1.05 \from\:both\:sides[/tex]
[tex]0.7y=3.5y-2.52[/tex]
[tex]0.7y-3.5y=3.5y-2.52-3.5y[/tex]
[tex]\mathrm{Subtract\:}3.5y\mathrm{\:from\:both\:sides} \nwarrow[/tex]
[tex]-2.8y=-2.52[/tex]
[tex]\frac{-2.8y}{-2.8}=\frac{-2.52}{-2.8} \hookleftarrow \mathrm{Divide\:both\:sides\:by\:}-2.8[/tex]
[tex]\boxed{\boxed{\underline{\textsf{\textbf{y=0.9}}}}}[/tex]
[tex]\bullet \bullet \bullet \bullet \bullet \bullet \bullet \bullet \bullet \bullet \bullet \bullet \bullet[/tex]
[tex]\textbf{HOPE IT HELPS}[/tex]
[tex]\textbf{HAVE A GREAT DAY!!}[/tex]
Chung has 6 trucks and 5 cars in his toy box. Brian has 4 trucks and 5 cars in his toy box.
Which is the correct comparison of their ratios of trucks to cars?
StartFraction 6 Over 4 EndFraction less-than StartFraction 5 Over 5 EndFraction
StartFraction 6 Over 4 EndFraction greater-than StartFraction 5 Over 5 EndFraction
StartFraction 6 Over 5 EndFraction less-than StartFraction 4 Over 5 EndFraction
StartFraction 6 Over 5 EndFraction greater-than StartFraction 4 Over 5 EndFraction
Given:
Chung has 6 trucks and 5 cars in his toy box.
Brian has 4 trucks and 5 cars in his toy box.
To find:
The correct comparison of their ratios of trucks to cars.
Solution:
The ratio of trucks to cars is defined as:
[tex]\text{Ratio}=\dfrac{\text{Number of trucks}}{\text{Number of cars}}[/tex]
Chung has 6 trucks and 5 cars in his toy box. So, the ratio of trucks to cars is:
[tex]\text{Ratio}=\dfrac{6}{5}[/tex]
Brian has 4 trucks and 5 cars in his toy box.
[tex]\text{Ratio}=\dfrac{4}{5}[/tex]
We know that,
[tex]6>4[/tex]
[tex]\dfrac{6}{5}>\dfrac{4}{5}[/tex]
Therefore, the correct option is D.
Answer:
what the guy above me said
Step-by-step explanation:
so yeah he is right points
How do you find the surface area
Answer:
It depends on what shape you have. Here are some formulas for different shapes.
Step-by-step explanation:
Rectangular prism: 2lw + 2lh + 2wh
Cylinder: 2 pi r² + 2 pi rh
Sphere: 4 pi r²
Cone: pi r² + pi rl
Square-based pyrimid: 1/2lp +B
I hope this helps!
Moving to another question will save this response.
1 points
Save Answer
Question 12
Mr Espent 65% of his salary on household expenses, and 15% of the remainder on travelling expenses and was finally left with R9 500. How much was his salary?
Answer:
rs.1680.67
Step-by-step explanation:
His salary = x
remaining % = 100 - 65 = 35%
= 100 - 15 = 85%
x × 35/100 × 85/100 = 500
x = 1680.67
Need help finding the factor of 2y^2-2y-4
Answer:
hope it helps you............
Answer:
2(y - 2)(y + 1)
Step-by-step explanation:
Given
2y² - 2y - 4 ← factor out 2 from each term
= 2(y² - y - 2) ← factor the quadratic
Consider the factors of the constant term (- 2) which sum to give the coefficient of the y- term (- 1)
The factors are - 2 and + 1, since
- 2 × 1 = - 2 and - 2 + 1 = - 1 , then
y² - y - 2 = (y - 2)(y + 1)
Then
2y² - 2y - 4 = 2(y - 2)(y + 1) ← in factored form
Joe drives for 3 hours and covers 201 miles. In miles per hour, how fast was he driving?
Answer:
67 mph
Step-by-step explanation:
201/3 = 67
Find the missing length indicated
x = 65
Step-by-step explanation:
cos theta = 25/x
cos theta = x/169
25/x = x/169
x² = 169 x 25
x = 65
The missing length x = 65, using the Pythagoras Theorem.
What is the Pythagoras Theorem?
According to the Pythagoras Theorem, in a right triangle, the square of the hypotenuse is equal to the sum of the squares of the legs.
How to solve the question?In the question, we are asked to find the value of x.
In the right triangle ABC, by Pythagoras' Theorem,
AC² + BC² = AB²,
or, x² + BC² = (144 + 25)²,
or, BC² = 169² - x² ... (i).
In the right triangle ACD, by Pythagoras Theorem,
AD² + DC² = AC²,
or, 25² + DC² = x²,
or, DC² = x² - 25² ... (ii).
In the right triangle BCD, by Pythagoras Theorem,
BD² + DC² = BC²,
or, 144² + x² - 25² = 169² - x² {Substituting BC² = 169² - x² from (i) and DC² = x² - 25² from (ii)},
or, x² + x² = 169² + 25² - 144² {Rearranging},
or, 2x² = 28561 + 625 - 20736,
or, 2x² = 8450,
or, x² = 4225,
or, x = √4225 = 65.
Thus, the missing length x = 65, using the Pythagoras Theorem.
Learn more about the Pythagoras Theorem at
https://brainly.com/question/231802
#SPJ2
Find the area of the following shape:
Answer:
36cm^2
Step-by-step explanation:
total area: 6x(4+3)=42
total area excluding the space: 42-(2x3)=36
Answer:
36 cm squared
Step-by-step explanation:
To solve this problem, I first construct a line. (shown in yellow in the first photo)
I then find the area of the top rectangle. (6 cm * 4 cm = 24 cm squared.)
Next, I find the area of the lower rectangle. But...to do that I have to find the length of the line that I constructed. To do this, I do 6cm-2cm=4cm.
Then I can find the area of the lower rectangle. (4cm*3cm=12cm squared.)
add up the area of both of the rectangles and.........12+24=36 cm squared
A person walks 1/6 mile in 1/18 hour.
The person's speed is _ miles per hour.
This Is What I Got!
Hope This Helps! :)
Have A Good Day!!
And If You Can I Wouldn't Mind A Brainliest! :))
Answer:
Divide 1/6 miles to 1/12hour since u wanna find our miles per hour
So it’ll be : 1/6 / 1/12
= 1/6 x 12/1
= 2 miles
For this question I am sure the answer is 81% as you divide 45 and 55. However, it is stating my answer is incorrect even though I put 0.81% as well. Did I round wrong or is the answer wrong completely?
Answer:
it says round to the nearest 10th so it wouldn't be 81, it would be 81.8%
B
These triangles
are congruent by
the triangle
congruence
postulate [?].
D
E
A. SSS
B. SAS
C. Neither, they are not congruent
Answer:
SAS
Step-by-step explanation:
AC ≅ EC (Given), ∠ACB ≅∠ECD ( Vertical Angles), and BC ≅ DC
Suppose that g(x)= f(x)+ 6. Which statement best compares the graph of g(x) with the graph of f(x)?
A. The graph of g(x) is the graph of f(x) shifted 6 units down.
B. The graph of g(x) is the graph of f(x) shifted to the right.
C. The graph of g(x) is the graph of f(x) shifted 6 units to the left.
D. The graph of g(x) is the graph of f(x) shifted 6 units up.
Answer:
D
Step-by-step explanation:
The + 6 moves it up 6 units.
The correct answer is (D) "The graph of g(x) is the graph of f(x) shifted 6 units up."
What is the function?A relationship between a group of inputs and one output is referred to as a function. In plain English, a function is an association between inputs in which each input is connected to precisely one output. A domain, codomain, or range exists for every function. Typically, f(x), where x is the input, is used to represent a function.
When we add a constant to a function, such as in the case of g(x) = f(x) + 6, it will shift the graph of f(x) upward by 6 units.
This is because, for any value of x, the value of f(x) will be added to 6, resulting in a vertical shift of the entire graph.
Option (A) is incorrect because adding 6 to f(x) would shift the graph up, not down.
Option (B) is incorrect because adding a constant to a function does not cause it to shift horizontally.
Option (C) is incorrect because adding 6 to f(x) would shift the graph right, not left.
D. The graph of g(x) is the graph of f(x) shifted 6 units up. Adding a constant term to a function will shift the graph of the function vertically. In this case, adding 6 to f(x) will shift the graph of f(x) upward by 6 units, resulting in the graph of g(x).
Learn more about function here:
https://brainly.com/question/29633660
#SPJ7
to train for a race, you plan to run 1 mile the first week and double the number of miles each week for five weeks. How many miles will you run for the 5th week. math problem
Answer:
16 Miles
Step-by-step explanation:
For every week you simply multiply the number of miles from the previous week by 2, therefore
Week 1: 1
Week 2: 2
Week 3: 4
Week 4: 8
Week 5: 16
Solve the equation 2x^2 + 3 – 41 = –15 to the nearest tenth.
Hellllpppp
9514 1404 393
Answer:
x = {-4.4, +2.9}
Step-by-step explanation:
We assume you want to solve ...
2x^2 +3x -41 = -15
Adding 41 and factoring out the leading coefficient gives ...
2(x^2 +3/2x) = 26
Dividing by 2 makes it ...
x^2 +3/2x = 13
We can add the square of half the x-coefficient to "complete the square."
x^2 +3/2x +(3/4)^2 = 13 +(3/4)^2
(x +3/4)^2 = 13.5625 . . . . write the left side as a square
x +3/4 = ±√13.5625 . . . . . take the square root
x = -0.75 ±3.683 = {-4.433, +2.933} . . . . subtract 3/4 and evaluate
The solutions are approximately x = -4.4 and x = 2.9.
21(2-y)+12y=44 find y
Answer:
[tex]\textbf{HELLO!!}[/tex]
[tex]21\left(2-y\right)+12y=44[/tex]
[tex]42-21y+12y=44[/tex]
[tex]~add ~similar\:elements[/tex]
[tex]42-9y=44[/tex]
[tex]Subtract~42~from~both~sides[/tex]
[tex]42-9y-42=44-42[/tex]
[tex]-9y=2[/tex]
[tex]Divide\:both\:sides\:by\:}-9[/tex]
[tex]\frac{-9y}{-9}=\frac{2}{-9}[/tex]
[tex]y=-\frac{2}{9}[/tex]
----------------------
hope it helps...
have a great day!
please help meeeeeeee
pt 4
Answer:
The answer is
[tex]2 {x}^{2} + 3x - 1 = 0[/tex]
Why? Below I explain
Step-by-step explanation:
That formula has three variables a, b and c.
So, a = 2, b = 3 and c = -1
Because the formula is written like
[tex] \frac{ - b + - \sqrt{ {b}^{2} - 4 \times a \times c} }{2 \times a} [/tex]
Find the distance between the pair of points: (0,1) and (1,0)
Answer:
sqrt(1^2 + 2^2)
[tex]\sqrt{2}[/tex]
Step-by-step explanation:
A road has a scale of 1:50 000 The length of a road on the map is 8.5cm.Work out the length of the real road in kilometres
Answer:
ok so
8.5*150000
1275000 cm into kilometers is
12.75 kilometers
Hope This Helps!!!
Alec bakes spherical rolls of bread. Each roll is about 8cm
wide. What is the approximate volume of each roll? Use
3.14 to approximate a.
Answer:
Step-by-step explanation:
2143.57
HELP ME PLEASE IF YOU DO YOU WILL GET BRAINLESS AND PLEASE EXPLAIN THE BEST YOU CAN
Answer:
<3=75°
Step-by-step explanation:
Angle 3 and angle 2x+95 are supplementary( supplementary angles add up to 180°)
So <3+2x+95=180
<3+2x=180-95
<3+2x=85( let's call this equation 1)
Next, angle 5 and angle 8x+71 are opposite angles (opposite angles are equal) therefore <5=8x+71
Now, <3 and <5 are co-interior angles(co-interior angles are supplementary)
So <3+8x+71=180
<3+8x=180-71=109
Thus, <3+8x=109(let's call this equation 2)
Now solving equation 1 and 2 simultaneously:
Make <3 the subject of equation 1
<3=85-2x
Put <3=85-2x into equation 2
85-2x+8x=109
6x=24
x=24/6=4
Now, remember that angle 2x+95 becomes
2(4)+95
8+95=103°
Therefore<3=180-105=75°
If using the method of completing the square to solve the quadratic equation x^2+15x+21=0, which number would have to be added to "complete the square"?
Step-by-step explanation:
the answer is in the above image
Answer:
my answer is 225/5 sorry comments for wrong